How to show that if $sum a_n^{3/2}$ is bounded implies $sum a_n/n$ is bounded?












0












$begingroup$



How to show that if $sum a_n^{3/2}$ is bounded implies $sum a_n/n$ is bounded?




Actually I do not able to compare both series .SO could not able to solve above problem?



What should be strategy to solve this problem?



ANy Help will be appreciated










share|cite|improve this question









$endgroup$

















    0












    $begingroup$



    How to show that if $sum a_n^{3/2}$ is bounded implies $sum a_n/n$ is bounded?




    Actually I do not able to compare both series .SO could not able to solve above problem?



    What should be strategy to solve this problem?



    ANy Help will be appreciated










    share|cite|improve this question









    $endgroup$















      0












      0








      0





      $begingroup$



      How to show that if $sum a_n^{3/2}$ is bounded implies $sum a_n/n$ is bounded?




      Actually I do not able to compare both series .SO could not able to solve above problem?



      What should be strategy to solve this problem?



      ANy Help will be appreciated










      share|cite|improve this question









      $endgroup$





      How to show that if $sum a_n^{3/2}$ is bounded implies $sum a_n/n$ is bounded?




      Actually I do not able to compare both series .SO could not able to solve above problem?



      What should be strategy to solve this problem?



      ANy Help will be appreciated







      real-analysis sequences-and-series






      share|cite|improve this question













      share|cite|improve this question











      share|cite|improve this question




      share|cite|improve this question










      asked Jan 18 at 13:08









      MathLoverMathLover

      52510




      52510






















          1 Answer
          1






          active

          oldest

          votes


















          2












          $begingroup$

          Try Hölder's inequality (assuming that $a_n$ is positive).






          share|cite|improve this answer









          $endgroup$









          • 1




            $begingroup$
            Is it mean that $sum |a_n/n|<sum |a_n^{3/2}||sum 1/n^3|^{1/3 }$ which is bounded
            $endgroup$
            – MathLover
            Jan 18 at 13:23












          • $begingroup$
            You lost a power of $frac{2}{3}$ for the first sum, but otherwise yes.
            $endgroup$
            – Klaus
            Jan 18 at 13:23












          • $begingroup$
            yes sorry I forget to write
            $endgroup$
            – MathLover
            Jan 18 at 13:27











          Your Answer





          StackExchange.ifUsing("editor", function () {
          return StackExchange.using("mathjaxEditing", function () {
          StackExchange.MarkdownEditor.creationCallbacks.add(function (editor, postfix) {
          StackExchange.mathjaxEditing.prepareWmdForMathJax(editor, postfix, [["$", "$"], ["\\(","\\)"]]);
          });
          });
          }, "mathjax-editing");

          StackExchange.ready(function() {
          var channelOptions = {
          tags: "".split(" "),
          id: "69"
          };
          initTagRenderer("".split(" "), "".split(" "), channelOptions);

          StackExchange.using("externalEditor", function() {
          // Have to fire editor after snippets, if snippets enabled
          if (StackExchange.settings.snippets.snippetsEnabled) {
          StackExchange.using("snippets", function() {
          createEditor();
          });
          }
          else {
          createEditor();
          }
          });

          function createEditor() {
          StackExchange.prepareEditor({
          heartbeatType: 'answer',
          autoActivateHeartbeat: false,
          convertImagesToLinks: true,
          noModals: true,
          showLowRepImageUploadWarning: true,
          reputationToPostImages: 10,
          bindNavPrevention: true,
          postfix: "",
          imageUploader: {
          brandingHtml: "Powered by u003ca class="icon-imgur-white" href="https://imgur.com/"u003eu003c/au003e",
          contentPolicyHtml: "User contributions licensed under u003ca href="https://creativecommons.org/licenses/by-sa/3.0/"u003ecc by-sa 3.0 with attribution requiredu003c/au003e u003ca href="https://stackoverflow.com/legal/content-policy"u003e(content policy)u003c/au003e",
          allowUrls: true
          },
          noCode: true, onDemand: true,
          discardSelector: ".discard-answer"
          ,immediatelyShowMarkdownHelp:true
          });


          }
          });














          draft saved

          draft discarded


















          StackExchange.ready(
          function () {
          StackExchange.openid.initPostLogin('.new-post-login', 'https%3a%2f%2fmath.stackexchange.com%2fquestions%2f3078229%2fhow-to-show-that-if-sum-a-n3-2-is-bounded-implies-sum-a-n-n-is-bounded%23new-answer', 'question_page');
          }
          );

          Post as a guest















          Required, but never shown

























          1 Answer
          1






          active

          oldest

          votes








          1 Answer
          1






          active

          oldest

          votes









          active

          oldest

          votes






          active

          oldest

          votes









          2












          $begingroup$

          Try Hölder's inequality (assuming that $a_n$ is positive).






          share|cite|improve this answer









          $endgroup$









          • 1




            $begingroup$
            Is it mean that $sum |a_n/n|<sum |a_n^{3/2}||sum 1/n^3|^{1/3 }$ which is bounded
            $endgroup$
            – MathLover
            Jan 18 at 13:23












          • $begingroup$
            You lost a power of $frac{2}{3}$ for the first sum, but otherwise yes.
            $endgroup$
            – Klaus
            Jan 18 at 13:23












          • $begingroup$
            yes sorry I forget to write
            $endgroup$
            – MathLover
            Jan 18 at 13:27
















          2












          $begingroup$

          Try Hölder's inequality (assuming that $a_n$ is positive).






          share|cite|improve this answer









          $endgroup$









          • 1




            $begingroup$
            Is it mean that $sum |a_n/n|<sum |a_n^{3/2}||sum 1/n^3|^{1/3 }$ which is bounded
            $endgroup$
            – MathLover
            Jan 18 at 13:23












          • $begingroup$
            You lost a power of $frac{2}{3}$ for the first sum, but otherwise yes.
            $endgroup$
            – Klaus
            Jan 18 at 13:23












          • $begingroup$
            yes sorry I forget to write
            $endgroup$
            – MathLover
            Jan 18 at 13:27














          2












          2








          2





          $begingroup$

          Try Hölder's inequality (assuming that $a_n$ is positive).






          share|cite|improve this answer









          $endgroup$



          Try Hölder's inequality (assuming that $a_n$ is positive).







          share|cite|improve this answer












          share|cite|improve this answer



          share|cite|improve this answer










          answered Jan 18 at 13:16









          KlausKlaus

          1,5349




          1,5349








          • 1




            $begingroup$
            Is it mean that $sum |a_n/n|<sum |a_n^{3/2}||sum 1/n^3|^{1/3 }$ which is bounded
            $endgroup$
            – MathLover
            Jan 18 at 13:23












          • $begingroup$
            You lost a power of $frac{2}{3}$ for the first sum, but otherwise yes.
            $endgroup$
            – Klaus
            Jan 18 at 13:23












          • $begingroup$
            yes sorry I forget to write
            $endgroup$
            – MathLover
            Jan 18 at 13:27














          • 1




            $begingroup$
            Is it mean that $sum |a_n/n|<sum |a_n^{3/2}||sum 1/n^3|^{1/3 }$ which is bounded
            $endgroup$
            – MathLover
            Jan 18 at 13:23












          • $begingroup$
            You lost a power of $frac{2}{3}$ for the first sum, but otherwise yes.
            $endgroup$
            – Klaus
            Jan 18 at 13:23












          • $begingroup$
            yes sorry I forget to write
            $endgroup$
            – MathLover
            Jan 18 at 13:27








          1




          1




          $begingroup$
          Is it mean that $sum |a_n/n|<sum |a_n^{3/2}||sum 1/n^3|^{1/3 }$ which is bounded
          $endgroup$
          – MathLover
          Jan 18 at 13:23






          $begingroup$
          Is it mean that $sum |a_n/n|<sum |a_n^{3/2}||sum 1/n^3|^{1/3 }$ which is bounded
          $endgroup$
          – MathLover
          Jan 18 at 13:23














          $begingroup$
          You lost a power of $frac{2}{3}$ for the first sum, but otherwise yes.
          $endgroup$
          – Klaus
          Jan 18 at 13:23






          $begingroup$
          You lost a power of $frac{2}{3}$ for the first sum, but otherwise yes.
          $endgroup$
          – Klaus
          Jan 18 at 13:23














          $begingroup$
          yes sorry I forget to write
          $endgroup$
          – MathLover
          Jan 18 at 13:27




          $begingroup$
          yes sorry I forget to write
          $endgroup$
          – MathLover
          Jan 18 at 13:27


















          draft saved

          draft discarded




















































          Thanks for contributing an answer to Mathematics Stack Exchange!


          • Please be sure to answer the question. Provide details and share your research!

          But avoid



          • Asking for help, clarification, or responding to other answers.

          • Making statements based on opinion; back them up with references or personal experience.


          Use MathJax to format equations. MathJax reference.


          To learn more, see our tips on writing great answers.




          draft saved


          draft discarded














          StackExchange.ready(
          function () {
          StackExchange.openid.initPostLogin('.new-post-login', 'https%3a%2f%2fmath.stackexchange.com%2fquestions%2f3078229%2fhow-to-show-that-if-sum-a-n3-2-is-bounded-implies-sum-a-n-n-is-bounded%23new-answer', 'question_page');
          }
          );

          Post as a guest















          Required, but never shown





















































          Required, but never shown














          Required, but never shown












          Required, but never shown







          Required, but never shown

































          Required, but never shown














          Required, but never shown












          Required, but never shown







          Required, but never shown







          Popular posts from this blog

          Mario Kart Wii

          What does “Dominus providebit” mean?

          Antonio Litta Visconti Arese